Solving the Laplace Transform of an Irreducible Quadratic Factor

Click For Summary
SUMMARY

The discussion focuses on finding the Laplace transform of the function \(\frac{(s+1)^2 + 6}{((s+1)^2 + 4)^2}\) using partial fractions. A participant suggests rewriting the numerator to simplify the expression, leading to the realization that the problem can be approached more effectively. The transformation involves separating the terms to facilitate the inverse Laplace transform, ultimately clarifying the solution process.

PREREQUISITES
  • Understanding of Laplace transforms
  • Familiarity with partial fraction decomposition
  • Knowledge of irreducible quadratic factors
  • Basic algebraic manipulation skills
NEXT STEPS
  • Study the properties of Laplace transforms
  • Learn about partial fraction decomposition techniques
  • Explore inverse Laplace transform methods
  • Investigate irreducible quadratic expressions in calculus
USEFUL FOR

Students and educators in engineering or mathematics, particularly those studying differential equations and control systems, will benefit from this discussion.

sunshine21
Messages
2
Reaction score
0

Homework Statement



i need to find the laplace transform of this equation by using the partial fraction. it is an irreducible quadratic factor that i don't really know how to solve it.

Homework Equations



[(s+1)^2 + 6]/[(s+1)^2 +4]^2

The Attempt at a Solution


please help me, thanks. =)
 
Physics news on Phys.org
Actually, you want the inverse Laplace transform.

Instead of using partial fractions, I think it might be helpful to rewrite the numerator this way:
\frac{(s + 1)^2 + 6}{((s + 1)^2 + 4)^2} = \frac{(s + 1)^2 + 4 + 2}{((s + 1)^2 + 4)^2} = \frac{(s + 1)^2 + 4}{((s + 1)^2 + 4)^2} + \frac{2}{((s + 1)^2 + 4)^2}
 
ouh..when you make it simple, now i can see how to solve it.
i didnt realize it. maybe i think to much on how to cancel out the square at the denominator.
thanks Mark44. your information is very helpful. thank you so much =))
 
Question: A clock's minute hand has length 4 and its hour hand has length 3. What is the distance between the tips at the moment when it is increasing most rapidly?(Putnam Exam Question) Answer: Making assumption that both the hands moves at constant angular velocities, the answer is ## \sqrt{7} .## But don't you think this assumption is somewhat doubtful and wrong?

Similar threads

  • · Replies 1 ·
Replies
1
Views
1K
  • · Replies 10 ·
Replies
10
Views
2K
Replies
9
Views
2K
  • · Replies 7 ·
Replies
7
Views
2K
  • · Replies 4 ·
Replies
4
Views
3K
Replies
2
Views
2K
  • · Replies 2 ·
Replies
2
Views
3K
  • · Replies 8 ·
Replies
8
Views
3K
  • · Replies 1 ·
Replies
1
Views
1K
  • · Replies 3 ·
Replies
3
Views
1K